Saltar al contenido principal
LibreTexts Español

9.5: Las ecuaciones de Maxwell en la forma 4

  • Page ID
    130539
  • \( \newcommand{\vecs}[1]{\overset { \scriptstyle \rightharpoonup} {\mathbf{#1}} } \)

    \( \newcommand{\vecd}[1]{\overset{-\!-\!\rightharpoonup}{\vphantom{a}\smash {#1}}} \)

    \( \newcommand{\id}{\mathrm{id}}\) \( \newcommand{\Span}{\mathrm{span}}\)

    ( \newcommand{\kernel}{\mathrm{null}\,}\) \( \newcommand{\range}{\mathrm{range}\,}\)

    \( \newcommand{\RealPart}{\mathrm{Re}}\) \( \newcommand{\ImaginaryPart}{\mathrm{Im}}\)

    \( \newcommand{\Argument}{\mathrm{Arg}}\) \( \newcommand{\norm}[1]{\| #1 \|}\)

    \( \newcommand{\inner}[2]{\langle #1, #2 \rangle}\)

    \( \newcommand{\Span}{\mathrm{span}}\)

    \( \newcommand{\id}{\mathrm{id}}\)

    \( \newcommand{\Span}{\mathrm{span}}\)

    \( \newcommand{\kernel}{\mathrm{null}\,}\)

    \( \newcommand{\range}{\mathrm{range}\,}\)

    \( \newcommand{\RealPart}{\mathrm{Re}}\)

    \( \newcommand{\ImaginaryPart}{\mathrm{Im}}\)

    \( \newcommand{\Argument}{\mathrm{Arg}}\)

    \( \newcommand{\norm}[1]{\| #1 \|}\)

    \( \newcommand{\inner}[2]{\langle #1, #2 \rangle}\)

    \( \newcommand{\Span}{\mathrm{span}}\) \( \newcommand{\AA}{\unicode[.8,0]{x212B}}\)

    \( \newcommand{\vectorA}[1]{\vec{#1}}      % arrow\)

    \( \newcommand{\vectorAt}[1]{\vec{\text{#1}}}      % arrow\)

    \( \newcommand{\vectorB}[1]{\overset { \scriptstyle \rightharpoonup} {\mathbf{#1}} } \)

    \( \newcommand{\vectorC}[1]{\textbf{#1}} \)

    \( \newcommand{\vectorD}[1]{\overrightarrow{#1}} \)

    \( \newcommand{\vectorDt}[1]{\overrightarrow{\text{#1}}} \)

    \( \newcommand{\vectE}[1]{\overset{-\!-\!\rightharpoonup}{\vphantom{a}\smash{\mathbf {#1}}}} \)

    \( \newcommand{\vecs}[1]{\overset { \scriptstyle \rightharpoonup} {\mathbf{#1}} } \)

    \( \newcommand{\vecd}[1]{\overset{-\!-\!\rightharpoonup}{\vphantom{a}\smash {#1}}} \)

    Este fondo formalista de 4 vectores ya es suficiente para analizar la transformada Lorentz del campo electromagnético. Sólo para calentarnos, consideremos la ecuación de continuidad (4.5),

    \[\ \frac{\partial \rho}{\partial t}+\nabla \cdot \mathbf{j}=0,\tag{9.110}\]

    que expresa la conservación de la carga eléctrica, y, como ya sabemos, es compatible con las ecuaciones de Maxwell. Si ahora definimos los 4-vectores contravariantes y covariantes de corriente eléctrica como

    \[\ \text{4-vector of electric current}\quad\quad\quad\quad j^{\alpha} \equiv\{\rho c, \mathbf{j}\}, \quad j_{\alpha} \equiv\{\rho c,-\mathbf{j}\},\tag{9.111}\]

    entonces la Ec. (110) puede representarse en la forma

    \[\ \text{Continuity equation: 4-form}\quad\quad\quad\quad \partial^{\alpha} j_{\alpha}=\partial_{\alpha} j^{\alpha}=0,\tag{9.112}\]

    mostrando que la ecuación de continuidad es invariante de forma 45 con respecto a la transformada de Lorentz.

    Por supuesto, la invarianza de una relación de tal forma no significa que todos los valores componentes de los 4 vectores que participan en ella sean los mismos en ambos fotogramas. Por ejemplo, tengamos alguna densidad de carga estática\(\ \rho\) en el cuadro 0; luego la Ec. (97b), aplicada a la forma contravariante del vector 4 (111), lee

    \[\ j^{\prime \alpha}=\frac{\partial x^{\prime \alpha}}{\partial x^{\beta}} j^{\beta}, \quad \text { with } j^{\beta}=\{\rho c, 0,0,0\}.\tag{9.113}\]

    Usando la forma particular (98) de la matriz recíproca de Lorentz para la elección de coordenadas mostrada en la Fig. 1, vemos que esta relación rinde

    \[\ \rho^{\prime}=\gamma \rho, \quad j_{x}^{\prime}=-\gamma \beta \rho c=-\gamma \nu \rho, \quad j_{y}^{\prime}=j_{z}^{\prime}=0.\tag{9.114}\]

    Dado que la velocidad de carga, como se observa desde el marco\(\ 0 \text {' }\)\(\ (-\mathbf{v})\), es, los resultados no relativistas serían\(\ \rho^{\prime}=\rho\),\(\ \mathbf{j}^{\prime}=-\mathbf{v} \rho\). El\(\ \gamma\) factor adicional en los resultados relativistas es causado por la contracción de longitud:\(\ d x^{\prime}=dx/\gamma\), de manera que para mantener\(\ d^{3} r=d x d y d z\) intacta la carga total\(\ d Q=\rho d^{3} r=\rho d x d y d z\) dentro del volumen elemental,\(\ \rho\) (y por lo tanto\(\ j_x\)) debe aumentar proporcionalmente.

    A continuación, al final del Capítulo 6 hemos visto que las ecuaciones de Maxwell para los potenciales electromagnéticos\(\ \phi\) y\(\ \mathbf{A}\) pueden ser representadas en formas similares (6.118), bajo el Lorenz (de nuevo, no “Lorentz”, ¡por favor!) condición de calibre (6.117). Para el espacio libre, esta condición toma la forma

    \[\ \nabla \cdot \mathbf{A}+\frac{1}{c^{2}} \frac{\partial \phi}{\partial t}=0.\tag{9.115}\]

    Esta expresión nos da una pista de cómo formar el 4-vector de los potenciales: 46

    \[\ A^{\alpha} \equiv\left\{\frac{\phi}{c}, \mathbf{A}\right\}, \quad A_{\alpha} \equiv\left\{\frac{\phi}{c},-\mathbf{A}\right\};\quad\quad\quad\quad\text{4-vector of potentials}\tag{9.116}\]

    de hecho, este vector satisface la Ec. (115) en su forma 4:

    \[\ \partial^{\alpha} A_{\alpha}=\partial_{\alpha}. A^{\alpha}=0\quad\quad\quad\quad\text{Lorenz gauge: 4-form}\tag{9.117}\]

    Dado que este producto escalar es invariante de Lorentz, y las derivadas (104) - (105) son 4 vectores legítimos, esto implica que el vector 4 (116) también es legítimo, es decir, obedece a las fórmulas de transformación de Lorentz (97), (99). Se puede obtener evidencia aún más convincente de este hecho a partir de las ecuaciones de Maxwell (6.118) para los potenciales. En el espacio libre, pueden ser reescritos como

    \[\ \left[\frac{\partial^{2}}{\partial(c t)^{2}}-\nabla^{2}\right] \frac{\phi}{c}=\frac{\rho c}{\varepsilon_{0} c^{2}} \equiv \mu_{0}(\rho c), \quad\left[\frac{\partial^{2}}{\partial(c t)^{2}}-\nabla^{2}\right] \mathbf{A}=\mu_{0} \mathbf{j}.\tag{9.118}\]

    Usando la definición (116), estas ecuaciones pueden fusionarse en una: 47

    \[\ \square A^{\alpha}=\mu_{0} j^{\alpha},\quad\quad\quad\quad\text{Maxwell equation for 4-potential}\tag{9.119}\]

    donde\(\ \square\) está el operador d' Alembert, 48 que puede representarse como cualquiera de dos productos escalares,

    \[\ \text{D’Alembert operator}\quad\quad\quad\quad \square \equiv \frac{\partial^{2}}{\partial(c t)^{2}}-\nabla^{2}=\partial^{\beta} \partial_{\beta}=\partial_{\beta} \partial^{\beta}.\tag{9.120}\]

    y por lo tanto es invariante de Lorentz. Debido a eso, y al hecho de que la transformada de Lorentz cambia ambos de 4 vectores\(\ A^{\alpha}\) y de\(\ j^{\alpha}\) manera similar, la Ec. (119) no depende de la elección del marco de referencia. Así hemos llegado a un punto clave de este capítulo: vemos que las ecuaciones de Maxwell son ciertamente invariantes de forma con respecto a la transformada de Lorentz. Como subproducto, la forma de 4 vectores (119) de estas ecuaciones (para potenciales) es extremadamente simple, ¡y hermosa!

    Sin embargo, como hemos visto en el Capítulo 7, para muchas aplicaciones las ecuaciones de Maxwell para los vectores de campo son más convenientes; así que representémoslas también en la forma 4. Para ello, podemos expresar todos los componentes cartesianos de los vectores vectores de campo habituales (3D) (6.7),

    \[\ \mathbf{E}=-\boldsymbol{\nabla \phi}-\frac{\partial \mathbf{A}}{\partial t}, \quad \mathbf{B}=\nabla \times \mathbf{A},\tag{9.121}\]

    vía los del potencial 4-vector\(\ A^{\alpha}\). Por ejemplo,

    \[\ E_{x}=-\frac{\partial \phi}{\partial x}-\frac{\partial A_{x}}{\partial t}=-c\left(\frac{\partial}{\partial x} \frac{\phi}{c}+\frac{\partial A_{x}}{\partial(c t)}\right) \equiv-c\left(\partial^{0} A^{1}-\partial^{1} A^{0}\right),\tag{9.122}\]

    \[\ B_{x}=\frac{\partial A_{z}}{\partial y}-\frac{\partial A_{y}}{\partial z} \equiv-\left(\partial^{2} A^{3}-\partial^{3} A^{2}\right).\tag{9.123}\]

    Completando cálculos similares para otros componentes de campo (o simplemente generándolos mediante cambios de índice apropiados), encontramos que el siguiente tensor de fuerza de campo antisimétrico y contravariante,

    \[\ F^{\alpha \beta} \equiv \partial^{\alpha} A^{\beta}-\partial^{\beta} A^{\alpha}\tag{9.124}\]

    puede expresarse a través de los componentes de campo de la siguiente manera: 49

    \ [\ F^ {\ alpha\ beta} =\ left (\ begin {array} {cccc}
    0 & -E_ {x}/c & -E_ {y}/c & -E_ {z}/c\\
    E_ {x}/c & 0 & -B_ {z} & B_ {y}\\
    E_ {y}/c & B_ {z} & 0 & -B_ {x}\\
    E_ {z}/c & -B_ {y} & B_ {x} & 0
    \ end {array}\ right),\ tag {9.125a}\]

    Tensores de fuerza de campo

    de manera que la forma covariante del tensor sea

    \ [\ F_ {\ alfa\ beta}\ equiv g_ {\ alfa\ gamma} F^ {\ gamma\ delta} g_ {\ delta\ beta} =\ left (\ begin {array} {cccc}
    0 & E_ {x}/c & E_ {y}/c & E_ {z}/c\\
    -E_ {x}/c & 0 & -B_ {z}/c & -B_ {z}/c & 0 & -B_ {z}/c & -B_ {} &
    B_ {y}\\ -E_ {y}/c & B_ {z} & 0 & -B_ {x}\\
    -E_ {z}/c & -B_ {y} & B_ {x} & 0
    \ end {array}\ right). \ tag {9.125b}\]

    Si la ecuación (124) parece demasiado voluminosa, tenga en cuenta que como recompensa, el par de ecuaciones Maxwell no homogéneas, es decir, dos ecuaciones del sistema (6.99), que en el espacio libre\(\ \left(\mathbf{D}=\varepsilon_{0} \mathbf{E}, \mathbf{B}=\mu_{0} \mathbf{H}\right)\) pueden reescribirse como

    \[\ \nabla \cdot \frac{\mathbf{E}}{c}=\mu_{0} c \rho, \quad \boldsymbol{\nabla}\times \mathbf{B}-\frac{\partial}{\partial(c t)} \frac{\mathbf{E}}{c}=\mu_{0} \mathbf{j},\tag{9.126}\]

    puede expresarse ahora de una manera muy simple (y manifiestamente invariante de forma),

    \[\ \partial_{\alpha} F^{\alpha \beta}=\mu_{0} j^{\beta},\quad\quad\quad\quad\text{Maxwell equation for tensor } F\tag{9.127}\]

    que es comparable con la Ec. (119) en su simplicidad — y belleza. Algo contra-intuitivamente, el par de ecuaciones homogéneas de Maxwell del sistema (6.99),

    \[\ \boldsymbol{\nabla} \times \mathbf{E}+\frac{\partial \mathbf{B}}{\partial t}=0, \quad \boldsymbol{\nabla} \cdot \mathbf{B}=0,\tag{9.128}\]

    mira, en la notación de 4 vectores, un poco más complicado: 50

    \[\ \partial_{\alpha} F_{\beta \gamma}+\partial_{\beta} F_{\gamma \alpha}+\partial_{\gamma} F_{\alpha \beta}=0.\tag{9.129}\]

    Obsérvese, sin embargo, que las ecuaciones (128) también pueden representarse en una forma mucho más simple de 4,

    \[\ \partial_{\alpha} G^{\alpha \beta}=0,\tag{9.130}\]

    usando el llamado tensor dual

    \ [\ G^ {\ alpha\ beta} =\ left (\ begin {array} {cccc}
    0 & B_ {x} & B_ {y} & B_ {z}\\
    -B_ {x} & 0 & -E_ {z}/c & E_ {y}/c\\
    -B_ {y} & E_ {z}/c & 0 & -E_ {x}/c\\
    -B_ {z} & -E_ {y}/c & E_ {x}/c & 0
    \ end {array}\ right),\ tag {9.131}\]

    que podrá obtenerse de\(\ F^{\alpha \beta}\), dada por la Ec. (125a), por las siguientes sustituciones:

    \[\ \frac{\mathbf{E}}{c} \rightarrow-\mathbf{B}, \quad \mathbf{B} \rightarrow \frac{\mathbf{E}}{c}.\tag{9.132}\]

    Además de la prueba de la invarianza de forma de las ecuaciones de Maxwell con respecto a la transformada de Lorentz, el formalismo de 4 vectores nos permite lograr nuestro objetivo inicial: averiguar cómo cambian los componentes del campo eléctrico y magnético en la transferencia entre (¡inercial!) marcos de referencia. Para ello, apliquemos al tensor\(\ F^{\alpha \beta}\) la transformada recíproca de Lorentz descrita por la segunda de las Eq. (109). Generalmente, da, para cada componente de campo, una suma de 16 términos, pero dado que (para nuestra elección de coordenadas, que se muestra en la Fig. 1) hay muchos ceros en la matriz de transformación de Lorentz, y los componentes diagonales de\(\ F^{\gamma \delta}\) igual cero también, los cálculos son bastante factibles. Calculemos, por ejemplo,\(\ E^{\prime}{ }_{x} \equiv-c F^{\prime 01}\). Los únicos términos distintos de cero en el lado derecho son

    \[\ E_{x}^{\prime}=-c F^{\prime 01}=-c\left(\frac{\partial x^{\prime 0}}{\partial x^{1}} \frac{\partial x^{\prime 1}}{\partial x^{0}} F^{10}+\frac{\partial x^{\prime 0}}{\partial x^{0}} \frac{\partial x^{\prime 1}}{\partial x^{1}} F^{01}\right) \equiv-c \gamma^{2}\left(\beta^{2}-1\right) \frac{E_{x}}{c} \equiv E_{x}.\tag{9.133}\]

    Repitiendo el cálculo para los otros cinco componentes de los campos, obtenemos relaciones muy importantes

    \ [\\ begin {array} {ll}
    E_ {x} ^ {\ prime} =E_ {x}, & B_ {x} ^ {\ prime} =B_ {x}\\
    E_ {y} ^ {\ prime} =\ gamma\ izquierda (E_ {y} -\ nu B_ {z}\ derecha), & B_ {y} ^ {\ prime} =\ gamma\ izquierda (B_ {y} +\ nu E_ {z}/c^ {2}\ derecha),\\
    E_ {z} ^ {\ prime} =\ gamma\ izquierda (E_ {z} +\ nu B_ {y}\ derecha), & B_ {z} ^ {\ prime} =\ gamma\ izquierda (B_ {z} -\ nu E_ {y}/c^ {2}\ derecha),
    \ end {array}\ tag {9.134}\]

    cuya forma “semi-vector” más compacta es

    \ [\\ texto {transformación Lorentz de componentes de campo}\ quad\ quad\ quad\ quad\ quad\ begin {array} {ll}
    \ mathbf {E} _ _ {|\ |} ^ {\ prime} =\ mathbf {E} _ {| |}, &\ mathbf {B} _ _ {|\ |} ^ {\ prime} =\ mathbf {B} _ _ {|\ |},
    \\\ mathbf {E} _ {\ perp} ^ {\ prime} =\ gamma (\ mathbf {E} +\ mathbf {v}\ veces\ mathbf {B}) _ {\ perp}, &\ mathbf {B} _ {\ perp} ^ {\ prime} =\ gamma\ izquierda (\ mathbf {B} -\ mathbf {v}\ veces\ mathbf {E}/c^ {2}\ derecha) _ {\ perp},
    \ end {array}\ tag {9.135}\]

    donde los índices\(\ \text { | | }\) y se\(\ \perp\) sitúan, respectivamente, para los componentes de campo paralelos y perpendiculares a la velocidad relativa\(\ \mathbf{v}\) de los dos marcos de referencia. En el límite no relativista, el factor Lorentz\(\ \gamma\) tiende a 1, y las ecuaciones (135) adquieren una forma aún más simple

    \[\ \mathbf{E}^{\prime} \rightarrow \mathbf{E}+\mathbf{v} \times \mathbf{B}, \quad \mathbf{B}^{\prime} \rightarrow \mathbf{B}-\frac{1}{c^{2}} \mathbf{v} \times \mathbf{E}.\tag{9.136}\]

    Así vemos que los campos eléctrico y magnético se transforman entre sí incluso en el primer orden de la\(\ \nu / c\) relación. Por ejemplo, si volamos a través de las líneas de campo de un campo uniforme, estático, puramente eléctrico\(\ \mathbf{E}\) (por ejemplo, el de un condensador plano) veremos no solo la renormalización del campo eléctrico (en el segundo orden de la\(\ \nu / c\) relación), sino también un campo magnético de CC distinto de cero\(\ \mathbf{B^{\prime}}\) perpendicular a tanto el vector\(\ \mathbf{E}\) como el vector\(\ \mathbf{v}\), es decir, a la dirección de nuestro movimiento. Esto es, por supuesto, lo que podría esperarse del principio de relatividad: desde el punto de vista del observador en movimiento (que es tan legítimo como el de un observador estacionario), las cargas superficiales de las placas del condensador, que crean el campo\(\ \mathbf{E}\), retroceden creando las corrientes dc (114), que inducen el campo magnético\(\ \mathbf { B' }\). Del mismo modo, el movimiento a través de un campo magnético crea, desde el punto de vista del observador en movimiento, un campo eléctrico.

    Este hecho es filosóficamente muy importante. Se puede decir que en la Madre Naturaleza no existe tal cosa como un campo eléctrico (o un campo magnético) por sí solo. No sólo el campo eléctrico puede inducir el campo magnético (y viceversa) en la dinámica, sino incluso en una configuración aparentemente estática, lo que medimos exactamente depende de nuestra velocidad en relación con las fuentes del campo —de ahí el término muy apropiado para todo el campo que estamos estudiando: el electromagnetismo.

    Otra aplicación simple pero muy importante de las ecuaciones. (134) - (135) es el cálculo de los campos creados por una partícula cargada que se mueve en el espacio libre por inercia, es decir, a lo largo de una línea recta con velocidad constante\(\ \mathbf{u}\), en el parámetro de impacto 51 (la distancia más cercana) b del observador. Seleccionando el marco de referencia 0' para moverse con la partícula en su origen, y el marco 0 para residir en el “laboratorio” en que\(\ \mathbf{B}\) se midan los campos\(\ \mathbf{E}\) y, podemos usar las fórmulas anteriores con\(\ \mathbf{v}=\mathbf{u}\). En este caso los campos\(\ \mathbf{E}^{\prime}\) y\(\ \mathbf{B}^{\prime}\) pueden calcularse a partir de, respectivamente, electro- y magnetostáticos:

    \[\ \mathbf{E}^{\prime}=\frac{q}{4 \pi \varepsilon_{0}} \frac{\mathbf{r}^{\prime}}{r^{\prime 3}}, \quad \mathbf{B}^{\prime}=0,\tag{9.137}\]

    porque en el marco 0', la partícula no se mueve. Seleccionando los ejes de coordenadas para que en el punto de medición\(\ x=0, y=b, z=0\) (Fig. 11a), para este punto podamos escribir\(\ x^{\prime}=-u t^{\prime}, y^{\prime}=b, z^{\prime}=0\), de manera que\(\ r^{\prime}=\left(u^{2} t^{\prime 2}+b^{2}\right)^{1 / 2}\), y los componentes cartesianos de los campos (137) sean:

    \ [\\ comenzar {reunido}
    E_ {x} ^ {\ prime} =-\ frac {q} {4\ pi\ varepsilon_ {0}}\ frac {u t^ {\ prime}} {\ left (u^ {2} t^ {\ prime 2} +b^ {2}\ derecha) ^ {3/2}},\ quad E_ {y} {\ prime} =\ frac {q} {4\ pi\ varepsilon_ {0}}\ frac {b} {\ left (u^ {2} t^ {\ prime 2} +b^ {2}\ derecha) ^ {3/2}},\ quad E_ {z} ^ {\ prime} =0,\\
    B_ {x} ^ {\ prime} =B_ {y} ^ {\ prime} =B_ {z} ^ {\ prime} =0.
    \ end {reunido}\ tag {9.138}\]

    Screen Shot 2022-03-01 a las 9.35.21 PM.pngFig. 9.11. Los pulsos de campo inducidos por una carga de movimiento uniforme.

    Ahora usando la última de las ecuaciones (19b) con\(\ x = 0\), dando\(\ t^{\prime}=\gamma t\), y las relaciones recíprocas a las ecuaciones (134) para la transformación de campo (es evidente que son similares a la transformación directa, con\(\ \nu\) reemplazadas por\(\ -\nu=-u\)), en el marco de laboratorio obtenemos

    \[\ E_{x}=E_{x}^{\prime}=-\frac{q}{4 \pi \varepsilon_{0}} \frac{u \gamma t}{\left(u^{2} \gamma^{2} t^{2}+b^{2}\right)^{3 / 2}}, \quad E_{y}=\gamma E_{y}^{\prime}=\frac{q}{4 \pi \varepsilon_{0}} \frac{\gamma b}{\left(u^{2} \gamma^{2} t^{2}+b^{2}\right)^{3 / 2}}, \quad E_{z}=0,\tag{9.139}\]

    \[\ B_{x}=0, \quad B_{y}=0, \quad B_{z}=\frac{n u}{c^{2}} E_{y}^{\prime}=\frac{u}{c^{2}} \frac{q}{4 \pi \varepsilon_{0}} \frac{\gamma b}{\left(u^{2} \gamma^{2} t^{2}+b^{2}\right)^{3 / 2}} \equiv \frac{u}{c^{2}} E_{y}.\tag{9.140}\]

    Estos resultados, 52 trazados en la Fig. 11b en las unidades de\(\ \gamma q^{2} / 4 \pi \varepsilon_{0} b^{2}\), revelan dos efectos principales. En primer lugar, el paso de carga por parte del observador genera no sólo un pulso de campo eléctrico sino también un pulso de campo magnético. Esto es natural, porque, como se discutió repetidamente en el Capítulo 5, cualquier movimiento de carga es esencialmente una corriente eléctrica. 53 En segundo lugar, las ecuaciones (139) - (140) muestran que la escala de duración del pulso es

    \[\ \Delta t=\frac{b}{n u}=\frac{b}{u}\left(1-\frac{u^{2}}{c^{2}}\right)^{1 / 2},\tag{9.141}\]

    es decir, se reduce a prácticamente cero a medida que la velocidad de la carga\(\ u\) se acerca a la velocidad de la luz Esto es, por supuesto, un corolario directo de la contracción relativista de la longitud: en el marco 0' que se mueve con la carga, la dispersión longitudinal de su campo eléctrico a\(\ b\) distancia de la línea de movimiento es del orden de\(\ \Delta x^{\prime}=b\). Cuando se observa desde el cuadro de laboratorio 0, este intervalo, de acuerdo con la Ec. (20), se reduce a\(\ \Delta x=\Delta x^{\prime} / \gamma=b/\gamma\), y por lo tanto también lo hace la escala de duración del pulso\(\ \Delta t=\Delta x / u=b / \gamma u\).


    Referencia

    45 En algunos textos, las ecuaciones que preservan su forma en una transformación se denominan “covariantes”, creando una posibilidad de confusión con los vectores y tensores covariantes. Por otro lado, llamar a tales ecuaciones “invariantes” no las distinguiría adecuadamente de las cantidades invariantes, como los productos escalares de 4 vectores.

    46 En las unidades gaussianas, el potencial escalar no debe dividirse por c en esta relación.

    47 En las unidades gaussianas, el coeficiente\(\ \mu_{0}\) en la Ec. (119) debe sustituirse, como de costumbre, por\(\ 4 \pi / c\).

    48 El nombre de Jean-Baptiste le Rond d'Alembert (1717-1783), quien ha hecho varias contribuciones pioneras a la teoría general de las olas —véase, por ejemplo, CM Capítulo 6. (Algunos libros de texto más antiguos usan notación\(\ \square^{2}\) para este operador.)

    49 En unidades gaussianas, esta fórmula, así como la Ec. (131) for\(\ G^{\alpha \beta}\), no tienen los factores c en todos los denominadores.

    50 Para ser justos, tenga en cuenta que al igual que la Ec. (127), la ecuación (129) esto también es un conjunto de cuatro ecuaciones escalares —en este último caso con los índices\(\ \alpha\)\(\ \beta\),, y\(\ \gamma\) tomando tres valores diferentes cualesquiera del conjunto\(\ \{0,1,2,3\}\).

    51 Este término es muy popular en la teoría de la dispersión de partículas — véase, por ejemplo, CM Sec. 3.7.

    52 En el siguiente capítulo, los volveremos a derivar de otra manera.

    53 Es sencillo usar la ecuación (140) y el principio de superposición lineal para calcular, por ejemplo, el campo magnético de una cadena de cargas que se mueve a lo largo de la misma línea y separadas por distancias iguales\(\ \Delta x=a\) (de manera que la corriente promedio, medida en el cuadro 0, es\(\ qu/a\)), y para muestran que el promedio de tiempo del campo magnético viene
    dado por la familiar Ec. (5.20) de magnetostáticos, con\(\ b\) en lugar de\(\ \rho\).


    This page titled 9.5: Las ecuaciones de Maxwell en la forma 4 is shared under a CC BY-NC-SA 4.0 license and was authored, remixed, and/or curated by Konstantin K. Likharev via source content that was edited to the style and standards of the LibreTexts platform; a detailed edit history is available upon request.